Você está na página 1de 32

Logical Reasoning

PDF compression, OCR, web optimization using a watermarked evaluation copy of CVISION PDFCompressor

Assumption Questions
Directions: The questions in this section are based on the reasoning contained in brief statements or passages. For some
questions, more than one of the choices could conceivably answer the question. However, you are to choose the best answer; that is, the response that most accurately and completely ans\.rr'ers the question. You should not make assumptions that are by commonsense standards implausible, superfluous, or incompatible with the passage.

Roberl

Speed limits on residential streets in Crownsbury are routinely ignored by drivers. People crossing those streets are endangered by speeding drivers, yet the city does not have enough police officers

2,

to patrol every street. So the city should install speed bumps and signs warning of their presence on residential streets to slow down traf8c.

Sheila: That is a bad idea.

People who are driving too

fast can easily lose control of their vehicles when they hit a speed bump.

1.
BO

Sheila's response depends on the presupposition that

Finnish author laakko Mikkeli was accused by Norwegian author Kirsten Halden of plagiarizing a book that she had written and that had been published 20 years before Mikkeli's. The two books, although set in different periods and regions, contain enough plot similarities to make coincidental resemblance unlikely. Mikkeli's defense rests on his argument that plagiarism was impossible in this case because Halden's book has been published only in Norwegian, a language Mikkeli does not understand, and because no reviews of Halden's book have ever been published.
The argument in Mikkeli's defense depends on

(A)

problems of the kind that Robert describes are worse in Crownsbury than they are in other
cities Robert's proposal is intended to address a problem that Robert does not in fact intend
address with speed bumps and warning signs in place,

the

'

assumption that

.=

tr

(B) (C) (D) (E)

(A) (B) iC) (D) (E)

Mikkeli has never met Halden


Halden's book did not become popular in

E
I

o UI o o
bto

it to

.g
J

IE

there would still be drivers who would not slow down to a safe speed most of the people who are affected by the problem Robert describes would be harmed by the installation of speed bumps and warning signs problems of the kind that Robert desoibes do not occut on any nonresidential streets in

Norway nobody related the plot of Halden's book in detail to Mikkeli before Mikkeli wrote his

book
there is a common European myth to which both authors referred subconsciously in the books in question Mikkeli is not familiar with Old lcelandic, an extinct language related to an earlier form of

Norwegian

Crownsbury

Sources: PrepTast 37, Sectton PrepTest 36, Sectlon

4, euestions 23 7, Questions 3

198

I(APLAN

PDF compression, OCR, web optimization using a watermarked evaluation copy of CVISION PDFCompressor

Assumption Questions

3.

Barnes: The two

ne\.vest employees at

this company

5.

Conservationist: The population ofa certain


wildflower is so small that the species is
headed for extinction. However, this wildflower can cross-pollinate with a closely related domesticated daisy, producing viable seeds. Such cross-pollination could result in a

have salaries that are too high for the simple tasks normally assigned to new employees and duties that are too complex for inexperienced

workers. Hence, the salaries and the complexity of the duties of these two newest employees should be reduced.

Which one of the following is an assumption on


which Barnes's argument depends?

(A) (l.i; ((.1


(ti
(E)
i

The duties of the two newest employees are not less complex than any others ia the company. it is because of the complex duties assigned that the two newest employees are being paid more than is usually paid to newly hired

significant population of wildflower-daisy hybrids. The daisy should therefore be introduced into the wildflower's range, since although the hybrid would differ markedly from the rvildflower, hybridization is the only means of preventing total loss of the wildflower in its range.

Which one of the following is an assumption on


which the conservationist's reasoning depends? (A)
(B) The wildflorver currently reproduces only by

'fhe fwo newest employees are not experienced at their occupations.


Barnes was not hired at a higher-than-average starting salary. The salaries of the two newest employees are no higher than the salaries that other

employees.

forming

seeds.

The domesticated daisy was bred from wild plants that once grew in the wildflower's
range.

(c)
(D)
(E)

companies pay for workers with a similar level of experience.

Increasing the population of the wildflower rvill also expand its range. Wildflower-daisy hybrids will be able tt' reproduce. The domesticated daisy will cross-pollinate with any daisylike plant.

F 0 q!,

4,

Political opinion and analysis outside the nrainstream rarely are found on television talk shows, and it might be thought that this state of affairs is a product ofthe political agenda ofthe television stations themselves. In fact, television stations are driven by the same economic forces as sellers of more tangible goods. Because they must attempt to capture the largest possible share of the television audience for their shows, they air only those shorvs that will appeal to large numbers of people. As a resultlpolitical opinions and analyses aired on television talk shows are typically bland and innocrious,
T'he explanation offered by the author of the passage makcs the assumption that

a
gl

Bacteria from food can survive for several days on the surface of plastic cutting boards, but bacteria can penetrate wooden cutting boards almost immediateln leaving the surface free of contamination. Therefore, wooden cutting boards, unlike plastic cutting boards, need not be washed in order to prevent their contaminating food that is cut on them; wiping them off to remove food debris is sufficient.

g!.

n G
|'l

o
=. = o(l

Which one of the following is an assumption on rvhich the argument depends?

(r\) (B) (C) (D) (E)

i.\
t.

j:i,

ii )
r,I--r

television station executives usually lack a political agenda of their own 'uland and innocuous political opinions and analyses are generally in the mainstream political analysts outside the mainstream are relatively indifferent to the effect their analyses have on television viewers most television viervers are prepared to argue against allowing the expression of political ophions and analyses with which they
disagree

Washing plastic cutting boards does not remove all bacteria from the surface. Prevention of bacteria contamination is the only respect in which wooden cutting boards are superior to plastic cutting boards. Food that is not already contaminated with bacteria can be contaminated only by being cut on contaminated cutting boards. Bacteria that penetrate into wooden cutting boards do not reemerge on the surface after the cutting boards have been used. Washing wooden cutting boards kills bacteria below the surface of the cutting boards.

(fl)

the political opinions of television station executives are not often reflected in the television shows their stations produce

Sources: PrepTest 29, Sectlon 7, Questlon 5 PrepTest 29, Section 4, Questlons 5 and 8 PrepTast 24, Sectlon 3, Questlon 5

E:9

lee

PDF compression, OCR, web optimization using a watermarked evaluation copy of CVISION PDFCompressor

ISAT Mostery Practice

7,

a method of producing text by means of raised dots that can be read by touch. A recent development in technology will allow flat conrputer screens to be made of a material that can be heated in patterns that replicate the patterns used in braille' Since the thermal device will utilize the same symbol system as braille, it follows that anyone who is accustomed to reading braille can easily adapt to the

Braille is

9.

Even though apes are the only nonhuman creatures able to learn human language, no ape has ever used its human language skill-q to ask such philosophical

questions as, "FIow am I different from all other creatures?" Therefore, philosophicai thought is unique to humans. The conclusion in the passage above relies on which oire of the following assumptions?

use of this electronic system.

Which one of the following is an assumption on rr'i:ich the conclusion depends? , .i j Braille is the only symbol system that can be . readily adapted for use with the new thermal
screen,

(A) (E )
(.--

Human language is unique to humans. Apes are incapable of thinking in human


Ianguage.

. '
bo

'
,

,= E .

' ,:

o (,l
G

Only people who currently use braille as their sole medium for reading text will have the capacity to adapt to the use of the thermal screen. People with the tactile ability to discriminate symbols in braille have an ability to discriminate similar patterns on a flat heated surface' Some symbol systems encode a piece of text by using dots that replicate the shape of letters of the alphabet. Eventually it will be possible to train people to read braille by first training them in the use of the thermal screen.

i (D) (Lr

Philosophical thought can be expressed only in human language. Speaking in human language is easier than thinking in human language. It is more difficult to learn human language than to express phil<isophical questions.

10.

A nerv medication for migraine seems effective, but there is concern that the medication might exacerbate heart disease. If patients with heart di-oease take the medication under careful medical supervision, howeverrharmfrrl side effects can definitely be avertedSilhe concern about those side ellects is thus unfounded.
,

-i

he argument depends on which one of the

8.

E
I

o
G

IT

I b0 o J

Itrrr:r:,1 with a financial crisis, Upland University's board of trustees reduced the budget for the unir,ersity's computer center from last year's $4 rnillion to $1.5 million for the coming year. However, the center cannot operate on less than $2.5 million' Silr,;e the board cannot divert funds from other pr'.si.tms to the computer center' there is no \^ray ti: rl rrri: center can be kept operating for the coming

ii-' il 1;11,in g

assumptions?

,.i-)
,

ij

i
.

The new medication actually is effective when taken by patients with heart disease. No migraine sufferers with heart disease will take the new medication except under careftrl medical supervision. Most migraine sufferers who have taken the new medication in trials also had heart
disease.

i*,'

j r,r
.,

i,rrclusion of the argument is properly drawn

if

(I]) (E)

, '
,
!

j.

l, one of the following is assumed? 'lhe computer center did not use all of the $4 million that was budgeted to it last year.
The budgets of other programs at the university were also reduced. fhe computer center has no source of funds other than those budgeted to it for the coming year by the university's board of trustees' No funds from any Program at the university can be diverted to other programs. The board oftrustees at the universityvalue other programs at the university more highly than they do the computer center.

The new medication has various other side effects, but none as serious as that of exacerbating heart disease. The new medication will displace all migraine medications currently being used.

i I )

, :.r

i'

Sources.'Preplest 22, Eectlon 2, Questlon Preplest 22, 9ectlon 4, Questlons


PrcpTest

!
2
and

2!,

lJ.

Sectlon

2, Questlon 3

200
PDF compression, OCR, web optimization using a watermarked evaluation copy of CVISION PDFCompressor

Assumption

9rlg!:

ll.

Copyright laws protect the rights of writers to profits earned from their writings, whereas patent laws protect inventors'rights to profits earned from their inventions. In Jawade, when computer-software writers demanded that their rights to profit be protected, the courts determined that information written for a machine does not fit into either the copyright or the patent category. Clearly, therefore, the profit rights of computer-software writers remain unprotected in lawade. Which one of the following is an assumption on which the argument depends?

13. Manager: Our new computer network,

the purpose of which is to increase productivity, can be installed during the day, which would disrupt our employees'work, or else at night, which ' would entail much higher installation charges. Since saving money is important, we should have the nefwork installed during me day.

The manager's argument assumes which one of the foliorving?


(A

) j

The monetary value of the network equipment rvould not exceed the cost of having the

(A)

equipment installed at night.


The monetary value of any productivity lost

iB) (C) (D) (E)

Computer-software writers are not an influential enough group in Jawade for the government to consider modifring existing copyright laws in order to protect this group's profit rights. No laws exist, other than copyright laws and patent laws, that would protect the profit rights of computer-software writers in
lawade.

i:i

1{_)

{D)

Most of the computer software used in Jawade is imported from other countries. Computer software is more similar to writings covered by copyright laws than it is to inventions covered by patent laws. Copyright laws and patent laws in Jawade have not been modified since their original adoption.

(E)
14.

during a daytime installation would be less than the difference between daytime and nighttime installation costs. A daytime installation would be completed by no larger a crew and would take the crew no more time than would a nighttime installation. Once the network has been installed, most of the companyt employees will be able to use it immediately to increase their productivity. Most of the company's employees would be able to work productively while a daytime installation is in progress.

fr

12.

One of the effects of lead poisoning is an inflammation of the optic nerve, which causes those who have it to see bright haloes around light sources. In order to produce the striking yellow effects in his "Sunflowers" paintings, Van Gogh used Naples yellow, a pigment containing lead. Since in his later paintings, Van Gogh painted bright haloes around the stars and sun, it is likely that he was suffering from lead poisoning caused by ingesting the

The symptoms of mental disorders are behavioral, cognitive, or emotional problems. Some patients with mental disorders can be effectively treated with psychotherapy. But it is now known that in some patients mental disorders result from chemical imbalances affecting the brain. Thus these patients can tre cffectively treated only with medication that rviil redr.tce or correct the imbalance. The argurnent depends on assuming which one of the foliorving?

E. f.t
Ol

g v o
a o
-

=. oe

(A) )

'lieatment by psychotherapy can produce no effective reduction in or correction of


chemical imbalances that cause mental disorders. I r:aatment with medication always shows faster rc'sults for patients with mental disorders than does treatment with psychotherapy. \lost mental disorders are not the result of ehemical imbalances affecting the brain. N{edication is always more effective in treating patient.s with mental disorders tlan is psychotherapy. teatment with psychotherapy has no efl'ect on mental disorders other than a reduction of the symptoms.

pigments he used. Which one of the following is an assumption on l.hich the argument relies?
(

(r\:
(B

((l) in'i (E)

In Van Gogh's later paintings he painted some things as he saw them. Van Gogh continued to use paints containing lead after having painted the "Sunflowers" paintings. Van Gogh did not have symptoms of lead poisoning aside from seeing bright haloes around light sources. The paints Van Gogh used in the "Sunflowers" paintings had no toxic ingredients other than
lead.

it i
{

([,;

The effects of Naples yellow could not have been achieved using other pigments.

Soutces,' PrepTest
PrepTest

21, Section 2, Questlon 6 21, Sectlon 3, Qusstlon 73

PrepTest 24, Sectlon PrcpTest 2O, Sectlon

!,

4, Question

Questlon 3

r(APrA9 2Ol
PDF compression, OCR, web optimization using a watermarked evaluation copy of CVISION PDFCompressor

ISAT Mastery Proctice


15.

John works five days each week except when on vacation or during weeks in rvhich national holidays occur. Four days a week he works in an insurance company; on Fridays he works as a blacksmith. Last week there were no holidays, and John was not on vacation. Therefore, he must have rvorked in the insurance company on Monday, Tuesday, Wedngsday, antl Thursday last week.

t7.

Many artists ciaim that art critics find it is easier to write about art that they dislike than to write about art that they like. Whether or not this hypothesis is correct, most art criticism is devoted to art works that fail to satisfy the critic. Hence it follows that rnost art criticism is devoted to rvorks other than the gieatest rvorks of art. 'l'ir,: r,r-.uclusion above is properly drawn if which

\\/high one of the following is an assumption on rlirich the argument depends?

rii

one

:r,-'

i rllol'ing is assumed? critic enjoys writing about art works rirat he or she dislikes intensely. i ii art critics find it difficult to discover art .,,'orks that truly satisfy them. ,'! ',r.ork of art that receives extensive critical attention can thereby become more widely . l'.noryn than it otherwise would have been. fhe greatest works of art are never recognized as -.uch until long after the time of their
:,.o atrt

(.,r.i
i.ll
i

' )

lohn never takes r.;eek in length.

vacation of more than one

i , : fl! )
{

) )

Every day last week that John lvorked, he rvorked for an entire rvorkday. J ohn does not take vacations in weeks in which national holidays occur. Last week John worked neither on Saturday

il

(E)

nor on Sunday. There were no days last week on which Iohn both worked in the insurance company and also worked as a blacksmith.
18.

creation.

thl

I'he greatest works of art are works that


inevitably satisfu all critics.

bto

16.

.=

tr

I,!

&
I

o o o
(E

Public health will improve more quickly in the wake of new medical discoveries if medical researchers abandon their practice of rvaiting until their findings are published in peer-reviewed journals before ilforming the press of important research results. This is because the public release of new medical information allows people to use that information in order to improve their health, but the peer-review process is unavoidably very slow.
\{,Ihich one of the following is an assumption on
u,,'1r:,"-lr

Ed'rrcational television is a contradiction in terms.


trVLile a classroom encourages social interaction, television encourages solitude. School is centered on the development of language, but television depends upon constantly changing visual images. And in a cla:;sroom, fun is merely a means to an end, but on teisi'ilion it is the end in itself.

tlic

.g
J

bo

iire argument depends?

llu,;rr rvirich one of the following assumptions does ::.',r.il;rx'rely in the passage? (,i i i ire classroom should not be a place where
( 1l

(A) ' 1i.;


(.
i
I

l'lany medical researchers do not agree to serve as reviewers when their own research in a critical phase.
Reviewers for many medical journals are not themselves medical researchers. People rvould use nerv medical information even if it were not first published in peer-

is
!.{

.i

i i

{i:;
i

(E)

revierved journals. 'Ihe peer-review process could be speeded up enough to produce a significant improvement in public health. New medical information that is first published in peer-revierved journals does not usually receive public attention.

i)

,lnvone has fun. )nly experiences that closely resemble what takes place in the school environrfient can be educational. "l-elevision programs reinforce some of the values of the school environment. E<iucational television programs are qualitatively better than most other television programs. -fhe potential
t

of television as a powerful learning tool has not yet been realized.

Sources.' PrepTest 2O, Section 4, Quesilon PrepTest !7, Section 2, Questlon t2 PrepTest 77, Sectlon 3, Questlon t4 PrepTest !O, Section Questlon 7

7j

t,

202

I(A

LAN

PDF compression, OCR, web optimization using a watermarked evaluation copy of CVISION PDFCompressor

Assumption Questions
19.

Sally: t cannot study at a university


university,

rvhere there is an alcohol problem, so unless something is done about the alcohol problem at this

2t. A favored theory to explain the extinction of


dinosaurs, together with many other species, has been the globally catastrophic collision of a large asteroid with the Earth. Supporti-ng evidence is an

I'll

have to transfer to a universin'

where there are no fralernities.

Yolanda: I dont agree that fraternities are


responsible for the alcohol problem at this university. Alcohol problems exist at all univ-ersities, including those rvhere there are no fraternities. We all should become more
aware of alcohol abuse. lt's not simply a fraternity problem; it's a cuitural problem.

Wliiclr orrc of the following is an assumption on

exlraterrestrial chemieal element in a layer of dust found rvorldwide at a geological level laid down conteilrporaneously with the supposed event. A nelv conrpeting theory contends $at any asteroid itnpaet was irrelevant, bec.ruse it rv#.nrassive volcirnic activity that cansed the extinctions by putting enough dust into the atmosphere to cool the planet )The l)eccan region of lndia contairrs extensive volcalic flolvs that occurred within the same time period as the supposed asteroid impact and the

n'lii;h rellv's argument

dePends?

,,,tinctions.
:
,

(Ar ( Ft)

1.irrr universities have fraternities.

hr

.rre rr.

theory assumes that

'
20,

''-;trthing rvill- be done about the alcohol l;r'oblem at Sally's university-. ((,, . t lcohol problems are becoming more - *'idespread at universities, {'i i r r lionre fraternity rnembers who drink alcohcl. bc'verages are too young to do so legally-' ii i - fhere could be univer.sities that have no

: ,
1

lJ,

(t
i-,

ihe massive volcanic activity was not caused bv the impact of an asteroid ,'<l individual dinosaurs survived the impact of the asteroid. if it occurred r he extinctions took place over a longer time period than they would have if caused by the inrpact of an asteroid

^ a?

I'i ,rfi and-burn agriculture involves burling several ,iL:.ri of forest, leaving vegetable ash that provides arr::rlc fslliliTer for three or four years of bountiful - l *ils, On the' cleared land nutrients leach out of the soii, horvever, and the land becomes too poor to sr.r!,.port agriculture. Netv land is then cleared by burning and the Process starts again' Since most frrrming in the tropics uses this method, forests in tlrr: rci;rr-rir rvill eventually be permanently
c

iJ

other lolcanic eruptions rvere not occurring ;rt O the same time as those in the Deccan region Oq it is not possibleto determine which would havc F'
occurred first, the volcanic flows in the Deccan Ot region or the supposed impact of an asteroid n

2)..

r.tdie itir"'tl.

'I-1rc
(

arq,rment depends on the assumption that


.lrriests in the tropics do not regenerate well lnough to restore themselves once they have been cleared by the slash-and-burn method cone other methods of egriculture are not as

i\)

()rrc of the requirements for admission to the O Lunnville Roller Skatrng Olub is a irisir degrce of skiU tt in roller skrrting. f'he club president has expressed 9! concern that th"e club rnayiave discriminaiecl against I qualified women in its admissions this year. Yet half of the applicants adrnitted to the club this year were =. 5 women. This proves that there was no discrimination OQ against qualified women applicants in the club's eclmissions this year.
\\rl i, h o ne of the following is an assumption on rvhich il'ri' , rr1,'lusion of the argtrment depends?
r

'(B

i (f, I ' llli (Fi

destructive to the environment in tropical regions as the slash-and-burn method is forests in the tropics are naturally deficient in nutrients that are needed to support the grorvth of plants that are not trative to those
regions

\
i

i;rdy

a few

applicants were lbund to be qualifiect

. ,

rrnd were admitted to the club this year.

{I

slash-and-burn agriculture is particularly suitable for farming in tropical areas slash-and-burn agriculture produces a more bountifirl crop than do other agriculture
.

ii:

'c more than half of all the roller skaters in I unnville are ll.omen. ''lo more than half of all the roller skaters in Lunnville are men. -[his year no more than half of the applicants who met all the qualifications for admission

methods for the iirst year

(I:.)

'fhis year no more than half of the rnembers ot


the club's committee that rnakes decisions about applicants' qualifications were men.

to the cJub rvere \{'omel{.

Sources: PrepTest 70, Sectlon !, Quastion 3 PrcpTest t0, Sectlon Quesfions / and 15 PrapTest 7O, Seetion 4, Question 4

l.

KAPLA9 .___/

2O3

PDF compression, OCR, web optimization using a watermarked evaluation copy of CVISION PDFCompressor

ISAT Mostery

Pradice 25,
The economies of some industrialized countries face the prospect of large labor shortages in the decades ahead. Meanwhile, these countries will have a vast number of experienced and productive older workers who, as things stand, will be driven from the work force upon reaching the age of sixty-five by the widespread practice of requiring workers to retire at that age. Therefore, if the discriminatory practice of mandatory'retirement at age sixty-five were eliminated, the labor shortages facing theseeconorlrii.r rvould be averted.

23.

Advertisement: Attention pond owners!


Ninety-eight percent of mosquito larvae in a pond die within minutes after the pond has been treated with BTI. Yet BTI is not toxic to fish, birds, animals, plants, or beneficial insects. So by using BTI regularly to destroy their larvae, you can greatly reduce populations of pesky mosquitoes that hatch in 7'our pond, and you can do so without diminishing the populations of fish, frogs, or beneficial insects in and around the pond. \A'hich one of the following is an assumption on r.,,hich the argument depends?

l hr. :it-grntent assumes

that

(.\l

'i

{A)
ili

(j)

he most effective way to control the numbers ,rf mosquitoes in a given area is to destroy the mosquito larvae in that area. ?opulations of mosquitoes are not dependent on a single body of water within an area as a place for their larvae to hatch and develop. 'lhere are no insect pests besides mosquitoes

ulLlci workers have acquired skills that are ' .i I remely valuable and that their younger ,'tileagues lack
'.:

i)ikers in industrialized countries are often

bo
II

tr tr

i11)\'

E
I

o o Itr o
tE 24.
EO

that pond owners might want to eliminate from in and around their ponds. -llhe effectiveness of BTI in destroying mosquito larvae in a pbnd does not require the pond owner's strict adherence to specific application procedures. The fish, frogs, and beneficial insects in and around a pond-owner's pond do not depend on mosquito larvae as an important source of food.

,ilr

(1

,,nprepared to face the economic ' ('nsequences of enforced idleness j,":ge number of workers in some ' i i r<lustrialized countries would continue x'orking beyond the age of sixty-five if workers in those countries were allowed to do so rrandatory retirement at age sixty-five lyas first instituted when life expectanry was considerably lower than it is today a substantial proportion of the population of officially retired rvorkers is actually engaged

in gainful employment

,g
J

'

English and the Austronesian language Mbarbaram both use the word "dog" for canines. These two languages are unrelated, and since speakers of the two languages only came in contact with one another long after the word "dog" lvas first used in this way in either language, neither language could have borrowed the word from the other. Thus this case shows that sometimes when languages share words that are similar in sound and meaning the similarity is due neither to language relatedness nor to borrorving. The argument requires that which one of the follo..ving be assumed? ' r.;

, :n

ii ' , ,

. . ,

ii-i) '
(t

English and Mbarbaram share no words other than "dog." Several languages besides English and Mbarbaram use "dog" as the word for canines. l.lsually when two languages share a word, ihose languages are related to each other. l here is no third language from which both English and Mbarbaram borrowed the word

"dog"'

If two unrelated languages share a word,


speakers of those trvo languages must have

come in contact with one another at some time.

Sources: PrepTest tO, Sectlon 4, Questlon 18 Preplest 9, Section 2, Questlon 27 PrepTest 9, Sectlon 4, Questlon 6

204
PDF compression, OCR, web optimization using a watermarked evaluation copy of CVISION PDFCompressor

Assumption Questions
26.

Myrna:

People should follow diets in which fat represents no more than 30 percent of total calories, not the 37 percent the average diet in this country contains.

28. Physiological

research has uncovered disturbing evidence linking a number of structural disorders to jogging. Among the ailments seemingly connected

'

with this now-popular sport are spinal disk


displaceme4ts, stress fractures of the feet and ankles, knee and hip joint deterioration, and tendonitis. Furthermore, these injuries do not occur exclusively among beginning runners-veteran joggers suffer an equal percentage of injuries. What the accumulating data suggest is that the human anatomy is not able to withstand the stresses of jogging.

Rolandr If everyone in the country followed your


recommendation during his or her entire life, just 0.2 percent would lenghen their lives at all' and then only by an average of 3 months. Modifying our diet is not worthwhile. A lifetime of sacrifice spent eating an unappealing low-fat diet is too high a price to pay for the chance of extending that sacrifice for 3 months.

N,[vrna: But for everyone who dies early from


such diets.

'".'irich one of the following is an assumption of the igumr:'ri?


iie lirrk between jogging and certain structural disorders appears to be a causal one. Iogging causes more serious disorders than other sports.
.

high-fat diet, many more people suffer from serious chronic diseases because they followed ;;:,. ,iid'-s iirgumrnt assumes that

.
:

."

, I

. l;

ri desirtble to live in such a way as to . icngthen life as much as possible .r lrrrv-fat diet cannot readily be made appealing and satisfying to a person who lollows it regularly dict is the only relevant factor to consider in computing influences on length of life the difference in tastiness between a diet in rvhich fat represents 30 percent of total calories and one in which it represents 37 percent is not noticeable not everyone in the country eats the average diet
is

iiJ) il-l
i;ntil

The jogger's level of experience is a factor determining the likelihood of a jogging injury. Some sports are safer for the human body than

jogging.

I'he human species is not very durable.

he rvas dismissed amid great controversy, Hastings was considered one of the greatest intelligence agents of all time. It is clear that if his dismissal was justified, then Hastings was either

F
f!

q3.

incompetent or else disloyal. Soon after the dismissal, however, it was shown that he had never been incornpetent. Thus, one is forced to conclude that
Hastings must have been disloyal. \\rh ir.lr one of the following states an assumption rr:.,.:r r,'hich the argument depends?
i

g
o g,
-l

n
t'l

27.

Some accountants calculate with simple

adding

"

inachines, and some use complex computers. One can perform more calculations in less time with a computer than with an adding machine. Therefore, assuming the costs of using the two types of machines are equal, an accountant who uses a computer generally can earn more per hour than an ?lccountant who uses an adding machine,

: .r, 'I

l\4rich one of the following is an assumption that lr.oukl make the conclusion in the passage a logical
-

i!)) (E)

ctt.
i.4

Hastings's dismissal was justified. Hastings was a high-ranking intelligence officer. The dismissal of anyone who was disloyal rvould be justified. Anyone whose dismissal was justified was disloyal. If someone was disloyal or incompetent, then his dismissal was iustified.

o 5

q(I =

' i :.,
r' )

More accountants use computers than use


adding machines. The more hours an accountant spends on the job, the more money he or she will earn. The more calculations an accountant performs, the more money he or she will earn. An accountant rvho uses an adding machine can charge a higher hourly rate than one who uses a computer. In general, accountants vary in terms of the number of calculations they make and the amount of monev thev earn.
Sources,' PrepTest

, I
"

lr)

9, Sectlon 4, Questlons 70 and 72

Preplest 6, Sectlon 2, Questlon 2 Preplest 6, Secfion 3, Questlon 9

205

PDF compression, OCR, web optimization using a watermarked evaluation copy of CVISION PDFCompressor

LSAT

Mastery Practice
32.

30

'llhe seventeenth-century physicist Sir Isaac Newton is remembered chiefly for his treatises on motion and gravity. But Newton also conducted experiments secretly for many years based on the arcane theories of alchemy, trying unsuccessfully to transmute common metals into gold and produce rejuvenating elixirs. If the alchemists of the seventeenth century had published the results of their experiments, chemistry in the eighteenth century would have been more advanced than it actually was' lArhich nne of the following assumptions would allow

The stated goal of the government's funding program for the arts is to encourage the creation ofworks of artistic excellence. Senator Beton claim3, however, that a government-funded artwork can never reflect the independent artistic conscience of the artist because artists, like anyone else who accepts financial support, will inevitably try to please those who control the distribution of that support. Senator Beton concludes that go\ernment funding of the afts not only is a burden on taxpayers but also cannot lead to the creation of works of true artistic
excellence.

the coqclusion concerning eighteenth-century chemistry to be properly drawn?

Which one of the following is an assumption on


rvhich Senator Beton's argument is based?

(A) lB) ill)


bo s
II

a 0
{a

tDl {B) lt.

|\' c, &,

Scientific progress is retarded by the reluctance ot historians to acknowledge the failures of sorne of the great scientists' i\dvances in science are'hastened when reports of experiments, whether successful or not, are available for review by other scientists. l'ien'ton's work on motion and gravity would nLlt have gained wide acceptance if the results of his work in alchemy had also been made nublic. Iircreasing specialization-within the sciences nrakes it difficult for scientists in one field to understand the principies of other fields. 'I'he seventeenth-century alchemists could have achieved their goals only if their experiments had been subjected to public scrutiny.
33.

(AJ l;lost taxpayers have little or no interest in the . creation ofworks oftrue artistic excellence. (it) Goyrnmot funding of the arts is more
generous than other financial support most

({''
(l )

artists receive.

ifistribution of government funds for the arts


is based oir a broad agreement as to what constitutes artistic excellence.

I )

Once an artist has produced works of true artistic excellence, he or she will never accept

( fr

government funding. A contemporary work of art that does not reflect the independent artistic conscience of the artist cannot be a work of true artistic
excellence.

-IE
trI

Annnals with a certain behavioral disorder have unusually high levels of aluminum in their brain tissue. Since a silicon-based compound binds to

bo

aluminum and prevents it from affecting the brain tissue, animals can be cured of the disorder by being treated rvith the comPound.
Tl're argument is based on which one of the following
ascr
{
?

rnptions?

School superintendent It is a sad fact that, until now, entry into the academically best high school in our district has been restricted to the children of people who were wealthy enough to pay the high tuition. Parents who were previously denied the option of sending their children to this school now have this option, since I am replacing the tuition requirement with a requirement that allows only those who live in the neighborhood of the school to attend. The superintendent's claim about the effect of repl.rcing the tuition requirement relies.on the

r\'}
ii

Animals with the disorder have unusually high but invariable levels of aluminum in their

r r1 ')
l

l:rain tissue.
Ai

rninum is the cause of the disorder rather tiran merely an effect of it. tniroducing the compound into the brain
tr

assumption that

(A)
: ir,

1) r

ii.csue has no side effects.

) t

the residents of the school's neighborhood tend to be wealthy people other than those wealthy enough to

{Il

['hc amount of the compound needed to nr'rltralize the aluminum in an animal's brain tissrre varies depending upon the species. i\lnrninum is never present in normal brain iissue,

(D) (E)

tuition are able to live in the neighborhood of the school people less wealthy than those who were able to pay the old tuition are in the majority in the district there are no high schools in the district other than the one referred to by the superintendent ' there are many people not wealthy enough to have paid the old tuition who wish to have their children attend the school
have paid the old

Sources.'Preprest 5, Section

Question 76 PrepTest 5, Sectlon 3, Questions 5,7, and 14

t,

206

I(APLAN

PDF compression, OCR, web optimization using a watermarked evaluation copy of CVISION PDFCompressor

Assumption Questions
34.

Although consciousttess seerns tc rrise fi'orn physical processes' physical tireories c;lr', r'1r)1;iiI! on[y why phi.sical systems ltirve ieit;ri,r "1r,,'ii,,tl :;tmctures and
horu these systelns pcrfornr \ r ,(,iis pirysical fune rions. Thus, uo stricf 1,. r-'i:r's1.',il theory can
e-r1; l;r

36. Editorialist:

rl

consciotrstress.

l-, :onclusion of thl .,;':rrilite rrt follorvs : ji one sf *ls fo|!ir,. ing is assumed? . ,

logically

if

Physical theories can explain only physical phenomena.

To ensure justice in the legal system, citizens must be capable of criticizing anyone involved in determining the punishment of criminals. But when the legal system's purpose is seen as deterrence, the system falls into the hands of experto whose specialty is to assess how potential lawbreakers are affected by the system's punishments, Because most citizens lack knorvledge about such matters, iustice is not then ensulctl in the legal system,

r,

I t.r

ii:i

An explanation of consciousness must encompass more than an explanation of physical structures and functions. The physical structures and functions of consciousness are currently unknown. Consciousness arises from processes that are entirely explainable by physical theories. An explanation ofphysical structures atrd functions must be formulated in strictly
physical terms.

fhe editorialist's argument requires assuming which


one of the following?

{.4,i

,.: '

35.

fr{) V

Cultural historian: Universal acceptance of scientific theories that regard human beings only as natural obiects subiect to natural forces outside the individualt control will inevitably lead to a general decline in morality. After all, if people do not believe that they are responsible for their actions, they rvill feel unashamed when they act iliinorallv, and a widespread failure of individuals - il t'eel ashamed of theiiimmoral actions is bound r,, lead to a general moral decline. i irr'.or,:lusion drawn by the cultural historian iriliir',','i icgi,:ally if rvhich one of the following is .: i - 'r :lt(l {

'
37.

Most citizens view justice as primarily concerned with the assignment of rrunishment to those who deserve it. Irr order to be just, a legal system must consider the effect that punishment will have t'n individual criminals. I he primary concern in a legal system is to ,:dminister punishments that are just. 1n a legal system, a concern for punishment is incompatible with an emphasis on deterrence. {iitizens without knowledge about how the iegal system's punishments affect potential lawbreakers are incapable of criticizing experts in that area,

fr'

F o q!.
ol I

The retail price of decaffeinated coffee is considerably higher than that of regular coffee. However, the process by which coffee beans are decaffeinated is fairly simple and not very co-stiy. Therefore, the price difference cannot be accor-rnted for by the greater cost of providing dei:affeinated coffee to the consumer. 'l'he argumcnt relies on assuming which one of the

o o o o I
J
lll

n
:'

follor,
(

rn!:?

i ,
:'1 .

Srrrrrce does not enable human beings to f r)Jrtrol natural forces,


i

ii,man beings who regard themselves only

as

) (l't)
.\

ij{ ()LL'ssing regular coffee costs more 1, r'ot:essing decaffeinated coffee.

than

oe

i;;'jcl differences between products can


srrrerally be accounted foi by such factors -:";ply and demand, not by differences in
as

l ) i

i)

latural objects rvill as a result lose their sense o1' responsibility for their actions. i'rople who have a sense of shame for their ;noral transgressions will feel responsible for iheir actions. 5ome scientific theories hold that human beings are not responbible for their actions.
Scientific explanations that regard human beings as in some respects independent of the laws of nature will not lead to a general decline in morality.

i, )

.;,Juction costs,
i , r ,r'1 i5
;
,

little competition among companies

,.

(1:

, '

,,;ii coffee-sellers do not expect that '!,sumers are content to pay more for ,rcaffeinated coffee than for regular coffee, i:e beans used for producing decaffeinated , offee do not cost much more before processing than the beans used for producing
regular coffee.

:, '

r {)focss decaffeinated coffee.

Sources: PrepTest 37, Section PrcpTest 37, Sectlon

2, 4, PrepTest 36, Section 7, PrepTest 36, Section 3,

Questton 5 Questlon 9 Questlon 20 Questlon 22

207

PDF compression, OCR, web optimization using a watermarked evaluation copy of CVISION PDFCompressor

LSAT

Mostery Praaice
40.

38

Novelists cannot become great as long as they'remarn in academia. Powers of observation and analysis, which schools successfully hone,.are useful to the novelist, but an inruitive grasp of the emotions of everyday life can be obtained only by the kind of immersion in everyday life that is precluded by being an academic.
.'rrl1irfu one of the following is an assumption on , ',lrh the argument depends?

'.
.)

, )

XT

{l:)
lo

)^(

Novelists require some impartiality to get an intuitive grasp of the emotions of everyday life. No great novelist lacks porvers of observation and iinalysis. Participation in life, interspersed with impartial observation of life' makes novelists great. Noveiists cannot be great without an intuitive grasp of the emotions of everyday life. Knowledge of the emotions of everyday life cannot be acquired by merely observing and analyzing life'

Political opinion and analysis outside the mainstream rarely are found on television talk shows, and it might be thought that this state of affairs is a product of the political agenda of the television stations themselves. In fact. television stations are driven by the same economic forces as sellers of more tangible goods. Because they must attempt to capture the largest possible share of the television audience for their shows, they air only those shows that will appeal to large numbers of people. As a result, political opinions and analyses aired on television talk shows are typically bland and innocuous.

An assumption made in the explanation offered by the author of the passage is that

iA)
('tj

.H@

E@ |n
rE

fo{arian Anderson,.the famous contralto, did not take success for granted. We know this because Anderson had to struggle early in life, and anyone who has tEr stmggle early in lit'e is ahle to keep a good r:r:rspective on the wcr'id.
'I
..'.

i,
r

''

) )

hc conclusion of the arllment follows logically , r ich one of the follorving is assumed?

if
,

&,
f,

o
|tr

'i : r: ''
:
'
r

r\lyone who succeeds takes success for granted.


.\nyone who is able to keep a good perspective on the world does not take success for granted. Anyone who is able to keep a good perspective on the world has to struggle early in life. ,.\nyone who does not take success for granted has to struggle early in life. Anyone who does not take success for granted is able to keep a good perspective on the world.

rnost television viewers cannot agree on which elements of a particular opinion or analysis arc most disturbing ttrere are television viewers who might refuse to watch television talk shows that they knew w'ould be controversial and disturbing < ach television viewer holds some opinion that is outside the political mainstream, but those opinions are not the same for everyone ihere are television shows on which'economic forces have an even greater impact than they do on television talk shows the television talk shows of different stations resemble one another in most respects

4t. i.,ommercial
a

rI

IJ aa

:!

) i

passenger airplanes can be equipped with collision-avoidance radar system that provides pilots rvith information about the proximity of other airplanes. Because the system warns pilots to take evasive action when it indicates a possible collision, passengers are safer on airplanes equipped with the . . system than on comparable airplanes not so equipped, even though the system frequently warns pilots to evade phantom airplanes.

Which one of the following is an assumption on


rvhich the argument depends?

(A)
{i-j

Passengers feel no saf*:

with the radar

sysit:t'r.

ln airplanes equipped than on comparable

) .,

i,

airplanes not so :,'r; rripped. Warnings given by a collision-avoidance system about phantom airplanes are not caused by distorted radar signals. The frequency of invalid warnings will not cause pilots routinely to disregard the
system's warnings.

D)

if,)
Sources: PrcpTest 35, Sectlon 7, Question 74 PrepTest 35, Sectron 4' questlon t4 Prcprest 29, Sectlon 4, Question 4 Prep.Test 26, Sectlon 2, Questlon 70

Commercial passenger airplanes are not the only planes that can be equipped with a collision-avoidance system The greatest safety risk for passengers traveling on commercial passenger airplanes is that of a midair collisios"

208

PDF compression, OCR, web optimization using a watermarked evaluation copy of CVISION PDFCompressor

@ @

42.

The Board of Trustees of the Federici Art Museum has decided to sell some works from its collection in order to raise the fulds necessary to refurbish its galleries. Although this may seem like a drastic remedy, the curator has long maintained that among thb paintings that the late Ms. Federici collected for the museum were several unsuccessful immature works by Renoir and C6zanne that should be sold becarrse they are of inferior quality and so add nothing to the overall quality of the museum's eoji.ir:tion. Hence, the board's action will not detract ficr:i :he quality of the museum's collection.-

43.

A certain credit-card company awards its customers bonus points for using its credit card. Customers can use accumulated points in the purchase of brand name merchandise by mail at prices lower than the manufacturers'suggested retail prices. At any given time, therefore, customers who purchase merchandise using the bonus points spend less than they rvould spend if they purchased the same merehandise in retail stores.

l{&ic}r one of the following is an assumption on ',rlt,,.ir ihe argument depends?

'flr*: ,..r:rclusion draulp depends on which one of the


foll
c',,,'':r

(,t., )

fhe merchandise that can be ordered by mail


using the bonus points is not offered at lower prices by other credit-card companies that award bonus points. The bonus points cannot be used by the creditcard customers in the purchase of brand name rnerchandise that is not available for purchase in retail stones. The credit-card company does not require its customers to accumulate a large number of tronus points before becoming eligible to order merchandise at prices lower than the manufacturers' suggested retail price. I'he amount credit-card customers pay for shipping the merchandise ordered by mail does not increase the amount customers spend to an amount greater than they rvould spend if they purchased the same merchandise in retail stores. fhe merchandise available to the company's credit-card customers using the bonus points is frequently sold in retail stores at prices that are higher than the manufacturers'suggested retail prices.

rg assumptioils?
ili

(1;
|

,'\rt speculators are unable to distinguish an inferior painting by Renoir from a


,nasterpiece bY him. 1l of the paintings that the board of trustees ;ells will be among those that the curator lecommends selling. all of the paintings by Renior and Czanne that are olvned by the Federici Art Museum were purchased by Ms. Federici herself. Only an avid collector of paintings by C6zanne would be rvilling to Pay a high price for early works by this artist. A great rvork of art can be truly appreciated only if it is displayed in a carefully designed and well-maintained gallerY.

!l'i
.i
r

i ,1

* *

'

;:

q3.

F
f'l

g
n o
Ol

t'l'
oq

0 = 3
II

Sources; PrcpTest 25, Sectlon PrepTest 20, Sectlon

4, Questlon 9 7, Questlon 77

209

PDF compression, OCR, web optimization using a watermarked evaluation copy of CVISION PDFCompressor

44.

Some people have been promoting a new herbal

45.

mixture as a remedy for the common cold. The mixture contains, among other things, extracts of the
plants purple coneflower and goldenseal. A cold sufferer, skeptical of the claim that the mixture is an effective cold remedy, argued, "Suppose that the mixture u'ere an effective cold remedy. Since most people with colds wish to recover quickly' it follows that almost everybody with a cold would be using it. Therefore, since there are many people who have colds but do not use the mixture, it is obviously not

Private industryis trying to attract skilled research scientists by offering them high salaries. As a result, most research scientists employed in private industry now earn 50 percent more than do comparably skilled research scientists employed by the government. So, unless government-employed research scientists are motivated more by a sense of public duty than by their own interests, the government is likely to lose its most skilled research scientists to private industry, since none of these scientists would have problems fi nding private-sector jobs,

effective." '
Each of the following is an assumption required by

lVhich one of the following is an assumption on


rvhich the argument depends?

the skeptical cold sufferer's argument EXCEPT:

(Ai

{r\) lB) ',{,) iD) (f,)

tk
bo

(B) (C)

g
E
IU

(t)) (E)

II

o o

Enough of the mixture is produced to provide the required doses to almost everybody with a cold. The mixture does not have side effects severe enough to make many people who have colds avoid using it' The mixture is powerful enough to prvent almost everybodY who uses it from contracting any further colds. The mixture is widely enough known that almost everybody with a cold is aware of it. There are no effective cold remedies available that many people who have colds prefer to the

mixture.

&
-IE tJ
II

cl

Government research scientists are less likely to receive acknowledgment for their research contributions than are research scientists in the private sector. None of the research scientists currently employed by the government earns more than the highest-paid researchers employed in the private sector. The government does not employ as many research scientists who are highly skilled as does any large company in the private sector which employs research scientists. The government does not provide its research scientists with unusually good working conditions or fringe benefits that more than compensate for the lower salaries they receive. Research scientists employed in the private sector generally work longer hours than do
researchers employed by the government.

0 J

blD

Sources.'Preplest 20, Sectlon 7, Questlon 20 Prepfest 2O, Sectlon 4, Questlon 76

210

I(APtAN

PDF compression, OCR, web optimization using a watermarked evaluation copy of CVISION PDFCompressor

Assumption Questions
46.

The current move to patent comPuter programs is a move in the wrong direction and should be stopped. The patent system was originally desigrred solely to protect small-time inventors from exploitation, not to give large corporations control over a methodology. Any computer program is merely the implementation of a methodology.

48.

Mary

Ann:

Our country should, above all, be strong. Strength gains the respect of other countries and makes a country admirable.
There are many examples in history of countries that were strong but used their strength to commit atrocities. We should judge a country by the morality of its actions, not by its strength. If the actions are morally good, the country is admirable.

lnez:

\'\&ich one of the following is an assumption on


rvhich the argument depends?

(-'\j
lF

r(l) (q) (E)


A1

Computer programs should be developed not only by large corporations but by small-time inventors as well. Implementing a methodology always requires less creative effort than does true invention. The issue of whether or not to patent computer programs Presents the patent system with problems that have never before
arisen. Large corporations should not hold patents for implementations of methodologies. Small-time inventors who support the move to patent comPuter programs act contrary to their own best interests.

\1*rich one of the following is a presupposition that underlies Inez' argument?

(r\) (B) (C) (D) (E)


49.

At least one country is admirable. Countries cannot be both strong and moral. It is possible to assign moral weight to the actions of countries. The citizens of any country believe that whatever their country does is good. Countries should impose their standards of morality on other countries by whatever
means nece$sary.

* *

J. J.

Scientists attempting to replicate certain controversial results reported by a group of experienced researchers failed to get the same results as those reported. The conclusion drawn from this by the scientists who conducted the replication experiments was that the originally reported results had been due to faulty measurements.

Thomson, the discoverer of the electron and a recipient of the Nobel Prize in physics, trained many physicists, among them seven Nobel Prize winners, 32 fellows of the Royal Society of London,pnd 83 professors of physics. This shows thaqftre skills peeded ior creative reiearch can be taught ancl)earned.)

l-

o(l

Which one of the following is an assumption on lvhich the argument depends?

(A) (B) i{l) {D) (E)

7 o
gl otl

n' !t r-r
n o 3 -r 3

J. J.

The argument of the scientists lvho conducted the


replication experiments assumes that

(A) (B) (C) (Ul (Fl)

the original experiments had not been described in sufficient detail to make an exact replication possitrle the fact that the originally reported results 'aroused controyersy made it highly likely that they were in error the theoretical principles called into question by the originally reported results were themselves based on weak evidence the replication experiments were not so likely as the original experiments to be marred by faulty measurements the researchers who originally reported the controversial results had themselves observed those results only once

Thomson was an internationally known physicist, and scientists came from all over the world to work with him. All the scientists trained by I. I. Thomson were renowned for their creative scientific r'esearch. At least one of the eminent scientists trained by I. I. Thomson was not a creative researcher before coming to study with him.
Creative research in physics requires research habits not necessary for creative research in other fields. Scientists who go on to be the most successfirl
researchers often receive their scientific

education in classes taught by renowned


research scientists.

Eources: PrepTeet 77, Sectlon 3, Questlon 3 fuepTest 9, Sectlon 4, Questlon 25 PrepTest 6, Sectlon 2, Questlons and

t5 211

I(APtAN

PDF compression, OCR, web optimization using a watermarked evaluation copy of CVISION PDFCompressor

50. Fire ants from Brazil now infest the soitl{ern

United

51. Head injury

States. Unlike queen fire ants in Brazil, two queens in the United States share a nest. Ants froni these nests are more aggressive than those from single-queen nests. By destroying virtually all insects in the nest area, these aggressive ants gain sole access to food sourcs, and the ant population skyrockets. Since

certain predator insects in Brazil limit the fue-ant population there, importing such predator insects into the United States would be of overall benefit to the environment by stopping the increase of the fire-ant population in the United States.
Each of the following is an assumption made in the

argument EXCEPT:

is the most serious type of injury sustained in motorcycle accidents. The average cost to taxpayers for medical care for nonhelmeted motorcycle-accident victims is twice that for their helmered counterparts. )urisdictions that have enacted rnotorcycle-helmet laws have reduced the incidence and severity of accident-related head injuries, thereby reducing the cost to taxpayers. Therefore, to achieve similar cost reductions, other jurisdictions should enact motorcycle-helmet laws. For the same reason jurisdictions should also require helmets for horseback riders, since horseback-riding accidents are even more likely to cause serious head injury than motorcycle accidents are.

(A)

* *
'
bo
tI

(*) ' {{:l (D) ' (E)

The imported insects would not prove more damaging to the environment in the United States than are the fire ants themselves. The predator insects from Brazil could survive in the ecological environment found in the

lrihich one of the following is an assumption upon


,,vlr;ch the author's conclusion concerning helmets iir', seback riders depends?

for

.'
i:

' '

United States.
The especially aggressive fire ants from the two-queen nests would not be able to destroy the Brazilian predator insects. The predator insects would stop the increase of the ant population before the ants spread to
states that are farther north.

Medical care for victims of horseback-riding accidents is a financial drain on tax funds. The higher rate of serious head injury suffered by victims of horseback-riding accidents is due to the difference in size betrveen horses
and motorcycles. The medical costs associated with treating head injuries are higher than those for other types of injury.

tr

i. :) i;.)

g
J

o o o o
bo

The rate ofincrease ofthe fire-ant population would not exceed the rate at which the predator insects could kill the ants.

Most fatalities resulting from horseback-riding and motorcycle accidents could have been prevented if the victims had been wearing
helmets.

E
o

When deciding whether to enact helmet laws for motorcyclists and horseback riders, the

.g

jurisdiction's primary concern is the safety of


its citizens.

52.

Twenty years ago the Republic of Rosinia produced nearly 100 million tons of potatoes, but last year the harvest barely reached 60 million tons. Agricultural researchers, who have failed to develop new higher-yielding strains of potatoes, are to blame for this dccrease, since they have been concerned only with thcir own research and not with the needs of Rosinia.

\\iiir; h one of the following is an assumption on


rvlri, lr the argument depends?
{,ri

.j;
r
Ij

i j

'
(

i',

Any current attempts by agricultural researchers to develop higher-yielding potato strains are futile, Strains of potatoes most commonly grown in Rosinia could not have produced the yields last year that they once did. Agricultural researchers often find concrete solutions to practical problems when investigating seemingly unrelated questions. fluctuations in the size of the potato crop ";\iide over a twenty-year period are not unusual. Agricultural research in Rosinia is funded by
goYernment grants.

Source: Prepfest 6, Sectlon

3, Questlons 3, 7, antl 75

212 SetA'9
PDF compression, OCR, web optimization using a watermarked evaluation copy of CVISION PDFCompressor

t
53.
Tbe reforms to improve the qualityof public education thet have been initiated on the part of suppliers of public education have been insufficient. Therefori,4pforms must be demanded by consumers. Parents should be given government vouc.hers with which to payfor their drildren's education and should be allowed to choose the schools at which the vouchers will be spent. To attract studenG, academically underachieving schools will be forced

Assumptian Questions

to improve their acadernic offerings.


The argument assumes that

,,

(A)
(B)

in selecting schools parents would tend to


prefer a reasonable level of ecadenic qudity to greatr sports oPportunities or more convenient location imprormnentin the acadenric ofieringp of schools willbe enforcdbythe discipline of the job market in which graduating students aompete there is a single best way to educate students children are able to recognize whic.h sctrools are better and would influence their parents' decisions
schools would each improve all of their

(c)
(D)
(E)

academic offeringp and wouldnot tend to specialize in one particular field to the exclusion of others

9ource; Prcptatt E, Sectlorr S, Queg,tlon

t0

{l[9

213

PDF compression, OCR, web optimization using a watermarked evaluation copy of CVISION PDFCompressor

|SAT Mastery Practice


54.

Literary

critic The meaning of a literary work is not fixed but fluid, and therefore a number of equally valid interpretations of it may be offered.
Interpretations primarily involve imposing meaning.ora-literary work rather than discovering meaning in it, so interpretations need not consider the writer's intentions, Thus' irny interpretation of a literary work tells more about the critic than about the writer.

56.

Editorial: The threat of harsh punishment for

\.{*'liich one of the following is an assumption

transgression usually decreases one's tendency to feel guilt or shame for committing that transgression, and the tendency to feel guilt or shame for committing a transgression reduces a person's tendency to commit transgressions, Thus, increasing the severity ofthe legal penalties for transgressions may amplify people's tendenry to ignore the welfare of others.

reiiuired by the literary critic's argument?

i.'

There are no criteria by which to distinguish


the validity of different interpretations

Y/hich one of the following is an assumption required by the editorial's argument?

of

(A)
(ij

>(
* *
u0
II

(li) i(lr
(Ii

()'I

tr
a-l

literary works. A meaning imposed on a literary work reflects facts about the interpreter. A writer's intentions are relevant to a valid interpretation of the writert work. The true intentions of the writer of a work of literature can never be known to a critic of that work. The deepest understanding of a literary work requires that one know the writer's history'

{(,) (D) (E)

Legal penalties do not determine the morality of an action. At least some actions that involve ignoring the welfare of others are transgressions. People who are concerned about threats to

their own well-being tend to be less


concerned about the welfare of others. The threat of harsh punishment deters people from committing transgressions only if this threat is at least sometimes carried out. Everyone has at least some tendency to feel guilt or shame for committing extrernely severe transgressions.

55.

&
tI

o UI G o
bo

Vague laws set vague limits on people's freedom, *'hich makes it impossible for them to know fgr certain whether their actions are legal, Thus,.irnder vague laws people cannot feel secure. ,

'i'he conclusion follows logically if which one of the i,',ilowing is assumed?

-tg lr,

r (Bl
(.4

if they know for certain whether their actions are legal. If people do not know for certain whether their actions are legal, then they might not
People can feel secure only feel secure.

We learn to use most of the machines in our lives through written instructions, without knor':ledge of the machines'inner workings, because mosl machines are specifically designed for use by nonexperts. So, in general, attaining technoit'gical expertise would prepare stude rrts for ttimorlo'rv's job market no better than rvould a more traditiunal education stressing verbal and quantitative skills. Thr, rrgument depenils on assurning rvhich one of thc r'ollowing?

(C

t (D) (E)

If people know for certain whether their


actions are legal, they can feel secure. People can feel secure ifthey are governed by laws that are not vague. Only people who feel secure can know for certain whether their actions are legal.

(A)
(Il

Fewer people receive a traditiorral education

stressing verbal and quantitative skiils norv

than did 20 years ago. Facility in operating machines designed for use by nonexperts is almost never enhanced by expert knowledge of the machines' inner
rvorkings.
N{ost jobs in tomorrorv's job market

(Cl
(

will not

{))

(!l)

demand the ability to operate many machines that afe designed for use only by experts. Students cannot attain technological expertise and also receive an education that does not neglect verbal and quantitative skills. When learning to use a machine, technological expertise is never more important than verbal and quantitative skills.

Sources: PrcpTest

37, Section 2, Question 75

Preplest 36, Sectlon 3, Question 72 PrepTest 37, Section 4, Questlon 75 Preplest 36, Sectlon 3, Questlon 74

214

I(APLAN

PDF compression, OCR, web optimization using a watermarked evaluation copy of CVISION PDFCompressor

Assumption Questions
58.

Herpetologist: Somepsychologistsattribute
complex reasoning to reptiles, claiming that simple stimulus-response explanations of some reptiles'behaviors, such as food gathering, cannot account fbr the complexity of such behavior. But sincc experiments show that reptiles are incapable of making major alterations in their behavior, for example, when faced u'ith significant changes in their environiilcnt, these animals must be incapabie of complex reasonirrg.

60.

In order to pressure the government of Country S to become less repressive, some legislators in Country R v/ant to ban all exports from R to S. Companies in R
that manufacture telecommunication equipment
such as telephones and fax machines have argued that exportg of their products should be exempted from the ban,.on the grounds that it is impossible for a country to remain repressive ra'hen telecommunication equipment is widely availabie to the population of that country.

Which one of the following is an assumption


required by the herpetologist's argument?

Which one of the following is an assumption on which the argument given by the manufacturers
depends?

{A)
ilJ)
/1-\

Animals could make major changes in t}reir behavior only if they rvere capable of complex reasoning. Simple stimulus-response explanations can in principle account for all reptile behaviors. Reptile behavior appears more complex in the field than laboratory experiments reveal it to
be.

(A)
(Il1

The government of S has recently increased the amount of telecomraurricatidn equipment __,,[it allorvs to be imported into the country. ) The telccommunication equipment that ivould b.e imported into S if the exemption were to frA be granted rvould not be available solely to

(C) )

(t))
i

If reptiles were capable of complex reasoning,


they would sometimes be able to make major changes in their behavior. Complex reasoning and responses to stimuli cannot both contribute to the same behavior.
a

top government officials in S. A majority of the members of R's legislature do not favor exempting telecommunication equipment from the ban on exports to

I
59.

(E)

(D

Country S. Of all exports that could be sent to Country

F
S,

common condition among mountain climbers, and one from which most people can recover, is caused by the characteristic shortage of orygen in the atmosphere at high altitudes. Cerebral edema, a rarer disruption of blood circulation in the brain that quickly becomes lifethreatening if not correctly treated from its onset, can also be caused by a shortage of orygen. Since the symptoms of cerebral edema gesemble those of

Ordinary mountain sickness,

rE\

telecommunication equipment would be the most effective in helping citizens of S oppose that country's repressive government.

oe II .||

g
n o

Without pressure from Country R, the government of S would be able to continue


repressing its citizens indefinitely.

ol
El

ordinary mountain sicknesslcerebral edema is rspet:ialiy dangerous at high )iltitudes.) \tiirich one of the following is an assumption on
rvliich tlie argument depends? (A; 'fhe treatment for ordinary mountain sickness differs from the treatment for cerebral edema. (ti; Cerebral edema can cause those who suf[er from it to slip into a coma within a few hours. i{l) Unlike cerebral edema, ordinary mouritain sicknebs involves no disruption of blood circulation in the brain. Shortage of oxygen at extremely high altitudes r l.l ) is likely to affect thinking processes and cause errors of judgment. (E) Most people who suffer from ordinary

o(I

o 3 lrt 5

mountain sickness recover without any


special treatment.

Sources: PrcpTest 36, Sectlon PrcpTest 29, Sectlon Preplest 27, Sectlon

t, Questlon 15 l, euesilon 2I I(API.AN) LIJ 'IE

3, Quesilon 76

PDF compression, OCR, web optimization using a watermarked evaluation copy of CVISION PDFCompressor

LSAT

Mastery Pradice
During recessions unemployment typically rises. Thus, during a recession air , pollution due to automobile exhaust decreases, silce fewer people commute in cars to jobs and so cars emitting pollutants into the ait are
used less.

'{1. Charles:

Alan:

Darlar

Why think that air pollution would

decrease?

During

a recession

fewer people can

afford to buy new cars, and cars tend to emit 'more pollutants as they get older.

Government subsidies have been proposed in Cariana to encourage farmers in Rochelle, the country's principal agricultural region, to irnplement certain new farming techniques, Urrless these techniques are implemented, erosion of productive topsoil cannot be coritrolled. Unfortunately, farmers cannot afford to shoulder the entire cost of the new tc,:hniques, which are more expensive than those

cnrrentiy ]:,:l;.1:'

r-rsed.

Therefore,

lrithout

subsidies,

.,*t

output in Rochelle rvill inevitablv

lVhich one of the following is an assumption on


r,,hich Charles's argument depends?
tA

Bettl'r

People who have never been employed drive no less frequently during a recession than

IJii'i er..osion in Rochelle is caused by lecuir'iiig flooris, r'.'hich will end next year once

l; )^( )^( r I' !

they rvould othenvise' Most air pollution is caused by automobile exhaust emitted by cars used by people commuting to jobs. idost people who are employed do not use any

Cariana corrrplr:tes the hydroelectric dam it is iruilding ;:-: o:;., the region's major river. 'fhereforr, il.r: irelle's total agricultural output lvill stabi:i2., ,-i its present level even without

ubsidie:..

!0 F
.I I

a-

-r

o UI
I

o g,
tu
II

ru

form of public transportation to commute to their jobs. During a recession, decreases in the use of cars resulting from reductions in commuting to iobs are not oft'set by increased use of cars for other reasons. During a recessiotr, a higher proportion of people who commute in cars to their jobs lose those jobs than do people who do not use cars to commute to their jobs.

1',;,r"i,
r'.

'i li' tty's argulilent


i,riilding
a

r rre

of tire f ,:i owing is an assumption on


depends?

(
:

i.

,' ,

dam across Rochellds major river will l,ut reduce any recurrent flooding that occurs 'r regions of Cariana other than Rochelle. , iie new farming techniques that must be i'nplemented to control soil erosion in Rochelle .rre not rvell suited to other regions of Cariana.
i ire

current yearly output, if any, from ll.ochelie's land that will be permanently ,rnder lvater once the dam is completed will at Jcast be matched by additional yearly output trom Rochelle's remaining land.

o J

uo

l-he cost to the government of Cariana to

operate the hydroelectric dam will not be greater than the projected cost of subsidizing the farmers of Rochelle in the implementation of the new farming techniques. 'lhe government of Cariana has sufficient financial resources both to subsidize its farmers' implementation of new farming techniques and to operate a hydroelectric dam,

Sources: fuepTest PrepTest

22, Section 2, Question !4 2!, Sectlon 2, Question 71

216

I(APLAN

PDF compression, OCR, web optimization using a watermarked evaluation copy of CVISION PDFCompressor

Assumption Quesfions
63. By examining fossilized beetles, a research team has produced the most detailed description yet of temperatures in Britain over the past 22'0fi) years. Fossils of species that still exist were selected and dated. When individuals of several species found in the same place were found to date to the same . period, the known temPerature tolerances of the species were used to determine the , i*itting beetle maximum summer temperature that could have existed at that place and Period. The procedure of the researchers assumes which one 65. A university should not be entitled to patent the inventions of its faculty members. Universities, as guarantors of intellectual freedom, should encourage the free flow of ideas and the general dissemination of knowledge. Yet a university that retains the right to patent the inventions cf its faculty members has a motive to suppress inforrnation about a potntially valuable discovery until the patent for it has been secured, Clearly, suppressing information concerning such discoveries is incompatible with the university's

obligation to promote the free flow of


arg,ument makes?

ideas.

of the following?

Which one of the following is an assumption that the

(A) (R) it-l) iI))

Beetles can tolerate warm weather better than

cold weather.
Fossils of different species found in the same place belonged to different period. The process of dating is more accurate for beetles than for other organisms. The highest actual summer temPerature at a place and period equaled the average of the" highest temPeraturs that could have been tolerated by each of the beetle species found there and dated to that Period' The temperature tolerances of the beetle species did not change significantly during

(A) (B) (C) D)


iii

Universities are the only institutions that have an obligation to guarantee intelleaual freedom. Most inventions by university faculty members wouid be profitable if patented.

Publication of reports on research is the only pra,.:tical way to disseminate information


crrncerning nerv discoveries.

* * *

(E)

the 22,000-year Period.

Uriversities that have a motiv to suppress information concerning discoveries by their l:,rculty members will occasionally act on that motive. If the inventions of a university faculty member are not patented by that university, then they will be patented by the faculty
member instead.

ce II
EL

o tl
tl

64, , X: Since many chemicals useful for agriculture and

medicine derive from rdre or endangered plant


species,

is likely that many plant species that are now extinct could have provided us with substances that rvould have been a boon to humanity. Therefore, if we want to ensure that chemicals from plants are available for use in the

it

future,l4e must make more serious efforts to preserve for all time our natural rebources'

Impact craters caused by meteorites smashing into Earth have been found all around the globe, but they have been found in the greatest density in geologically stable regions. This relatively greater abundance of securely identified craters in geologically stable regions must be explained by the lower rates ofdestructive geophysical processes in
those regions. The conclusion is properly drawn if which one of the following is assumed?

n o
gl

=. = oe

Y: But living things

are not our "resources." Yours is a selfish approach to conservation, We should rather strive to preserve living species because they deserve to,survive, not because of the good

they can do us' X's argument relies on which one of the following assumptions?

(A) (B) (C) (D) (E)

iA) (B) (C) (D) (E)

Medicine would now be more advanced than it is if there had been a serious conservation policy in the past. All living things exist to serve humankind. The use ofrare and endangered plant species as a source for chemicals will not itself render those species extinct. The only way to persuade people to preserve natural re$ources is to convince them that it is in their interest to do so. Fes if any, plant species have been saved from extinction through human efforts.

A meteorite that strikes exactly the same spot as an earlier meteorite will obliterate all traces of the earlier impact. Rates of destructive geophysical processes within any given region vary markedly throughout geological time. The rate at which the Earth is struck by
meteorits has greatly increased in geologically recent times. Actual meteorite impacts have been scattered fairly evenly over the Earth's surface in the 'course of Eartht geological history. The Earth's geologically stable regions have been studied more intensively by geologists than have its less stable regions.

Sources: Prcplaat

17, Sectlon 8, Questton 25 Prcpleil 70, Sectlon 4, Quesuon 8 Preplest 9, Sectlon 2, Quesilon 19 Prcplcst 5, Sectlon I, euesilon lll

r(APrA9 217

PDF compression, OCR, web optimization using a watermarked evaluation copy of CVISION PDFCompressor

ISAT Mastery

Pradice

67.

Being articulate has been equated with having alatge vocabular,v. Actually, however, people r,r'ith large vocabularies have no incentive for, and tend not to engage in, the kind of creative linguistic selfexpression that is requrired when no available words seem adequate. Thus a large vocabulary is a hindrance to using language in a truly articulate way.

Which one of the following is an assumption made


the *rgument?

il

(A) iBi ' {C) (U; (E)

l4'l-ren people are truly articulate, they have the capacity to express themselves in situations in which their vocabularies seem inadequate' People who are able to express themselves creatively in new situations have little

* * *
bo g t:

tr

incentive to acquire large vocabularies. The most articulate people are people who have large vocabularies but also are able to express themselves creatively when the situation demands it. In educating people to be more articulate, it r.;'ould be futile to try to increase the size of their vocabularies. ln unfamiliar situations, even people with large vocabularies often do not have specifically suitable words available'

a ul
c,

rg

* -IE
T-

v
bo

o J

Soutce,' Preplest 5, Sectlon

3, Questlon

t7

.)
L

1 tv

Q I(APIAN)
.-___/

PDF compression, OCR, web optimization using a watermarked evaluation copy of CVISION PDFCompressor

Assumption Questions
68.

The desire for praise is the desire to obtain, as a sign that one is good, the favorable opinions of others, But because people merit praise only for those actions motivated by a desire to help others, it follows that one who aids others primarily out of a desire for praise does not deserve praise for that aid.

70.

Large-scale government projects designed to benefit everyone-such as roads, schools, and bridgesusually benefit some small segments of society, initially at least, more than others. The more

Which one of the following, if assumed, enables the conclusion of the argument to be properly drawn?

(A) (B) (C) (D) (E)

An action that is motivated by a desire for the favorable opinion of others cannot also be
motivated by a desire to help others. No action is worthy of praise if it is motivated
solely by a desire for praise. People who are indifferent to the welfare of others do not deserve praise. One deserves praise for advancing one's olvn interests only if one also advances the

equally and widely political power is distributed among the citizenry, the less likely such projects are to receive funding. Hence, government by referendum rather than by means of elected representatives tends to diminish, not enhance, the welfare of a society.

Which one of the following is an assumption on


which the argument depends?

(A) (B) (C) (D) (E)

Large-scale government projects sometimes enhance the welfare of societv, Large-scale projects are more likely to fulfill their intended purposes if they are not executed by the government.

It

interests ofothers. is the motives rather tllan the consequences of one's actions that determine whether one 'deserves praise for them.

Government by referendum actually undermines the democratic process. The primary purpose of an equal distribution of political power is to enhance the welfare of
society.

* * * *

69.

Physician: Hatha yoga is a powerful tool for helping people quit smoking. In a clinical trial, those who practiced hatha yoga for 75 minutes once a week and received individual counseling reduced their smoking and cravings for tobacco as much as did those who went to traditional self-help groups once a week and
had individual counseling.

Government by referendum is the only way to distribute political power equally and widely.

71. Moderate exercise lowers the risk of blockage of the


arteries due to blood clots, since anything that lowers blood cholesterol levels also lowers the risk of hardening of the arteries, which in turn lowers the risk of arterial blockage due to blood clots; and, if the data reported in a recent study are correct, moderate exercise lowers blood cholesterol levels.

E. fr
ql lrf

E 7 o
o l!. 3

Which one of the following is an assumption on which the physician's argument relies?

(A) (B) (C) (D) (E)

The individual counseling received by the smokers in the clinical trial who practiced hatha yoga did not help them quit smoking. Most smokers are able to practice hatha yoga

The conclusion drawn above follows logically which one of the following is assumed?

if

(A) (B) (C) (D) (E)

more than once

a week.

Traditional self-help groups are powerfril tools for helping people quit smoking. People who practice hatha yoga for 75 minutes once a week are not damaging themselves
physically.

Other forms of yoga are less effective than hatha yoga in helping people quit smoking.

The recent study investigated the relationship between exercise and blood cholesterol levels. Blockage ofthe arteries due to blood clots can be prevented. Lowering blood cholesterol levels lowers the risk of blockage of the arteries. The data reported in the recent study are correct. Hardening of the arteries increases the risk of blockage ofthe arteries due to blood clots.

oq

Sources,' PrcpTost 37, Sectlon PrepTeat 37, teetion Preplest 36, Sectlon

2, Question t9 4, Questions t9 and 2O

!,

Questlon

!8

{tP

21e

PDF compression, OCR, web optimization using a watermarked evaluation copy of \ CVISION PDFCompressor

In the paintings by seventeenth-century Dutch artist Vermeer, we find several recutrent items: a satin iacket, a certain Turkish carpet, and wooden chairs with lion's head finials. These reappearing objects might seem to evince a dearth of props. Yet we know that many of the props Vermeer used were expensive' Thus, while we might speculate about exactly why Vermeer worked with a small number of familiar objects, it was clearly not for lack of props that the recurrent items were used'
The conclusion follows logically if which one of the following is assumed?

74. It is said that people should accept themselves as they are instead of being dissatisfied with their own abilities. But this is clearly a bad principle if the goal
is a society whose citizens are gertuinely happy,

for

no one can be genuinely happy if he or she is not pursuing personal excellence and is unwilling to undergo personal change of any kind.

Which one of the following is an assumption


required try the argument?

(A) (B) (C) (D) (E)


/J,

(A)

* * * *
UE
II

(B) (C) (D) (E)

Vermeer often borrowed the expensive props he represented in his paintings. The props that recur in Vermeer's paintings were always available to him. The satin jacket and wooden chairs that recur in the paintings were owned byVermeer's
sister.

Those who are willing to change will probably find genuine happiness. People who are not dissatisfied with themselves are less likely than others to pursue personal excellence. Personal excellence cannot be acquired by those who lack genuine confidence in their

own abilities.
People are justified in feeling content with themselves when they have achieved some degree of personal excellence. Happiness is not genuine unless it is based on something that is painftrl to obtain.

The several lecurrent items that appeared in Vermeer's paintings had special sentimental

fi

tr

importance for him. Ifa dearth ofprops accounted for the recurrent objects in Vermeer's paintings' we would not see expensive props in any of them.

!o .E@
,ill
"u0 ,Ol

8 'r.

s@

Any writer whose purpose is personal expression sometimes uses words ambiguously. Every poet's purpose is personal expression. Thus no Poetry reader's enjoyment depends on attaining a precise understanding of what the poet means.
The conclusion can be properly inferred if which one of the following is assumed?

The current pattern of human consumption of resources, in which we rely on nonrenewable resources, for example metal ore, must eventually change. Since there is only so much metal ore available, ultimately we must either do without or turn to renewable resources to take its place.

Which one of the following is an assumption


required by the argument?

(A) (B) (C) (D) (E)

(A) (B) (C) (D) (E)

Writers who sometimes use words ambiguously have no readers who try to attain a precise understanding of what the

writer means.
Writers whose purpose is personal expression are unioncerned with tvhether anyone enjoys
reading their works. No writer who ever uses words ambiguously has any reader whose enjoyment depends on attaining a precise understanding of what the

There are renewable resource replacements for all of the nonrenewable resources currently being consumed. We cannot indefinitely replace exhausted nonrenewable resources with other nonrenewable resources. A renewable resource cannot be exhausted by

human consumption. Consumption of nonrenewable resources will not continue to increase in the near future. Ultimately we cannot do without
nonrenewable resources.

writer means.
Most writers whose readers'enjoyment does not depend on attaining a Precise understanding of the writers'words are poet$. Readers who have a precise understanding of what a writer has written derive their enjoyment from that understanding.

Sources: Prepfest 36, Section 7, Questlons 26 and 22 PrepTest 36, Section 3, Questlon 78 Prcpfest 35, Sactlon !, Queetlon !8

220

I(APtAN

PDF compression, OCR, web optimization using a watermarked evaluation copy of CVISION PDFCompressor

Assumption Questions
Columnist: Almost anyone
can be an expert, for there are no of6cial guidelines determining what an expert must know. Anybody who manages to convince some people of his or her qualifications in an area-whatever those may be-is an expert.

78.

Publicity campaigns for endangered species are unlikely to have much impact on the most important
environmental problems, for while the ease of attributing feelings to large mammals facilitates evoking sympathy for them, it is more difficult to elicit sympathy for other kinds of organisms, such as the soil microorganisms on which large ecosystems and agriculture depend. Which one of the following is an assumption on which the argument depends?

The columnist's conclusion follows logically if which one of the following is assumed?

(A) (B) (C) (D) (E)

Almost anyone can convince some people of his or her qualifications in some area. Some experts convince everyone of their qualifications in almost every area. Convincing certain people that one is qualified in an area requires that one actually be qualified in that area. Every expert has convinced some people of his or her qualifications in some area' Some people manage to convince almost everyone of their qualifications in one or
more areas,

(A) (B) (C) (D) (E)


79.

@ @

77.

No chordates are tracheophytes, and all members of Pteropsida are tracheophyles' So no members of Pteropsida belong to the family Hominidae.
The conclusion above follows logically if which one of the following is assumed?

The most important environmental problems involve endangered species other than large mammals. Microorganisms cannot experience pain or have other feelings. Publicity campaigns for the environment are the most effective when they elicit sympathy for some organism. People ignore environmental problems unless they believe the problems will affect creatures with which they sympathize. An organism can be environmentally significant only if it affects large ecosystems or agriculture.

* * * *

Several critics have claimedthat any contemporary

(A)
(B)

All members of the family Hominadae are


tracheophytes.

All members of the family Hominidae are


chordates.

poet who writes formal poetry-poetry that is rhymed and metered-is performing a politically conservative act. This is plainly false. Consider Molly Peacock and Marilyn Hacker, two contemporary poets whose poetry is almost exclusively formal and yet who are themselves politically progressive feminists. The conclusion drawn above follows logically which one of the following is assumed?

(c)
(D)
(E)

All tracheophytes are members of Pteropsida. No members of the family Hominidae are
chordates.

if

No chordates are members of Pteropsida.

(A) (B) (C) (D) (E)

No one who is a feminist is also politically


conservative.

No poet who writes unrhymed or unmetered poetry is politically conservative. No one who is politically progressive is capable of performing a politically conservative act. Anyone who sometimes writes poetry that is not politically conservative never writes poetry that is politically conservative. The content of a poet's work, not the work s form, is the most decisive factor in determining what political consequences, if any, the work will have.

Sources: PrepTest PrepTest PrepTest PrepTest

35, Sectlon

35, Sectlon
35,

!, Questlon 2O t, Questlon 22 35, Sectlon 4, Questlon t8


S*tlon 4, Questlon 79

f$ry

221

PDF compression, OCR, web optimization using a watermarked evaluation copy of CVISION PDFCompressor

LSAT

Mostery Practice
the capacity

80. Ethicist: Studies have documented

of

81.

placebos to reduce pain in patients who believe that they are receiving beneficial drugs. Some doctors say that they administer placebos because medically effective treatment reinforced by the placebo effect sometimes helps patients recover faster than good treatment alone. But administering placebos is nonetheless ethically questionable, for even if a placebo benefits a patient, a doctor might, for example, have prescribed it just to give the patient satisfaction

The price of a firll-fare coach ticket from Toronto to Dallas on Breezeway Airlines is the same today as it was a year ago, if inflation is taken into account by calculating prices in constant dollars, However, today 90 percent of the Toronto-to-Dallas coach tickets that Breezeway sells are discount tickets and only 10 percent are fi.rll-fare tickets, rvhereas a year ago half were discount tickets and half were fuIl-fare tickets. Therefore, on average, people pay less today in constant dollars for a Breezeway Toronto-to-Dallas coach ticket than they did a year ago.

that something was being done.

*r
* * *
bo tr rr

The ethicist's argument depends on which one of the following assumptions?

Which one of the following, if assumed, would allow the conclusion above to be properly drawn?

(A) (B) (C) (D) (E)

(A) (B) (C) (D) (E)

e a tu o g
I

a-

A patient's psychological satisfaction is not a consideration in administering medical treatment. The motivation for administering a placebo can be relevant to the ethical justification for doing so. Medical treatment that relies on the placebo effect alone is ethically indefensible. The pain relief produced by the placebo effect justifies the deception involved in administering a placebo. Administering a placebo is not ethically justified if that treatment is not prescribed by a doctor,

A Toronto-to-Dallas full-fare coach ticket on Breezeway Airlines provides ticket-holders with a lower level of service today than such ticket provided a year ago. A Toronto-to-Dallas discount coach ticket on Breezeway Airlines costs about the same amount in constant dollars today as it did a
year ago.

All full-fare coach tickets on

Breezeway
as

Airlines cost the same in constant dollars

.g

ru

they did a year ago. The average number of coach passengers per flight that Breezeway Airlines carries from Toronto to Dallas today is higher than the average number per flight a year ago. The criteria that Breezeway Airlines uses for permitting passengers to buy discount coach tickets on the Toronto-to-Dallas route are different today than they were a year ago.

o J

bto

Sources.' PrepTest 33, Sectrbn PtepTest 29, Sectlon

!,

Questlons

t3

7, Questlon 2O

222

PDF compression, OCR, web optimization using a watermarked evaluation copy of CVISION PDFCompressor

82. Historian:

We can learn about the medical history of individuals through chemical analysis of their hair. It is likely' for example, that Isaac Newton's psychological problems were due to mercury poisoning; traces of mercury were found in his hair. Analysis is now being done on a lock of Beethoven's hair. Although no convincing argument has shown that, Beethoven ever had a venereal disease, some people hypothesize that venereal disease caused his deafness. Since mercury was commonly ingested in Beethoven's time to treat venereal disease, if researchers find a trace of mercury in his hair, we can conclude that this hypothesis is correct,

83. Monica:

The sculpture commissioned for our town plaza has been scorned by the public ever since it went up. But since the people in our town do

not know very much about contemporary art, the unpopularity of the work says nothing about its artistic merit and thus gives no reason for removing it.

Hector: You may be right about what the


sculpture's popularity means about its artistic merit. However. a work of art that was commissioned for a public space ought to benefit the public, and popular opinion is ultimately the only way of determining what the public feels is to its benefit. Thus, if public

opinion of this sculpture is what you say, then

Which one of the following is an assumption on which the historian's argument depends?

it certainly ought to

be removed.

(A) (B) (C) (D) (E)

None of the mercury introduced into the body


can be eliminated. Some people in Beethoverls time did not

The argument Hector makes in responding to Monica depends on the assumption that

(A)

ingest mercury. Mercury is an effective treatment for venereal


disease.

Mercury poisoning can cause deafness in


people with venereal disease. Beethoven suffered from psychological problems of the same severity as Newtont.

(B)

(c)
(D)
(E)

no matter what the public's opinion is on an issue affecting the public good, that public opinion ought to be acted on, eyell though the opinion may not be a knowledgeable one Monica's assessment of the public's opinion of the sculpture is accurate if the sculpture had artistic merit, then even a public that was not knowledgeable about modern art would not scorn the sculpture worlis of art commissioned for public spaces ought not to be expected to have artistic merit if the public feels that it does not benefit from the sculpture, this shows that the public does not in fact benefit from the sculpture

* * * *

F
l'l

q3.

g v o
Ol

{fi

84.

The dwarf masked owl, a rare migratory bird of prey, normally makes its winter home on the Baja peninsula, where it nests in the spiny cactus. In fact, there are no other suitable nesting sites for the dwarf masked owl on the Baja peninsula. But a blight last spring destroyed all of the spiny cacti on the Baja peninsula. So unless steps are taken to reestablish the spiny cactus population, the dwarf masked owl will not make its home on the Baja peninsula this winter. The argument depends on assuming which one

oq

o :I =.

of

the following?

(A) (B) (C) (D)


Sowces: PrcpTest 28, Sectlon 3, Questlon 76 fueplest 26, Sectlon 2, Questlon 25 Preplest 25, Sectlon 4, Question 25 (E)

No birds of prey other than the dwad masked owl nest in the spiny cactus. If the Baja peninsula contains spiny cacti, then the dwarf masked owl makes its winter honre
there.

On occasion the dwarf masked owl has been known to make its lvinter bome far from its normal migratory route. The dwarf masked orvl will not make its winter home on the Baja peninsula only if that region contains no spiny cacti. Suitable nesting sites must be present where the dwarf masked owl makes its winter home.
I(APLAN

223

PDF compression, OCR, web optimization using a watermarked evaluation copy of CVISION PDFCompressor

E5.

The widespread staff reductions in a certain region's economy are said to be causing people whostill have their jobs to cut back on ne\{i purchases as though they, ioo, had become economically distressed' Clearly, however, actual spending by such people is undiminished, because there has been no unusual increase in the amount of money held by those people in savings account'

87.

Dinosaur

* * * *
b0

Which one of the following is an assumption on which the argument relies? (A) If people in the region who continue to be emplbyed have debts' they are not now paying them off at an accelerated rate. (B) People in the region who continue to be employed and who have relatlves who have losi their jobs commonly assist those relatives

erpert: Some paleontologists have claimed that birds are descendants of a group of dinosaurs called dromeosaurs. They appeal to the fossil record, which indicates that dromeosaurs have characteristics more similar to birds than do most dinosaurs. But there is a fatal flaw in their argument; the earliest bird fossils that have been discovered date back tens of millions of years farther than the oldest known dromeosaur fossils. Thus the paleontologists' claim is false'
Having similar characteristics is not a sign that types of animals are evolutionarily related. Dromeosaurs and birds could have common
ancestors . Knowledge of dromeosaur fossils and the earliest bird fossils is complete. Known fossils indicate the relative dates of origin of birds and dromeosaurs. Dromeosaurs are dissimilar to birds in many

The expert's argument depends on assuming which one of the following?

(A) (B) (C) (D) (E)


88.

(C) (D)

financiallY. If people in the region who have lost jobs get nirv iobs, the new jobs generally pay less well

than the ones theY lost. People in the region who continue to be

rT

,S

(E)
86.

s E
tt tg
{9,

employed are pessimistic about thei'r prosPects for increasing their incomes' fhere exist no statistics about sales ofgoods in the region as a whole.

significant ways.
Spectroscopic analysis has revealed the existence of frozen nitrogen, methane, and carbon monoxide on the surface of Pluto. Such ices have a tendency to vaporize, producing an atmosphere. Since the proportion of any gas in such an atmosphere depends directly on how readily the corresponding ice vaporizes, astronomers have concluded that the components of Pluto's atmosphere are nitrogen, carbon monoxide, and methane, in order of decreasing abundance. The astronomers'argument relies on which one the followin g assumptions?

x
r:

E
J

ru

rJ

!o

Professor Robinson: A large meteorite impact crater in a certain region was thought to be the clue to explaining the mass extinction of plant and animal species that occurred at the end of the Mesozoic era' However, the crystalline structure of rocks recoveted at the site indicates that the impact that formed this crater luas not the culprit. When molten rocks crystallize' they display the polarity of Earth's magnetic field at thai time' But the recrystallized rocls recovered at the site display normal magnetic polarity, even though Earth's magnetic field was reversed at the time of the mass extinction'
Each of the following is an assumption on which

of

(A) (B) (C) (D) (E)

Professor Robinson' s argument depends EXCEPT:

(A) (B) (C) (D) (E)

The crater indicates an impact of more than sufficient size to have caused the mass

extinction.
The recovered rocks recrystallized shortly after they melted. No other event caused the rocks to melt after the imPact formed the crater. The recovered rocks nelted as a result of the impact that formed the crater. The mass extinction would have occurred soon after the impact that supposedly caused it.

There is no more frozen nitrogen on the surface of Pluto than there is either frozen carbon monoxide or methane. Until space probes reach Pluto, direct analysis of the atmosphere is impossible. There is no frozen substance on the surface of Pluto that vaporizes more readily than methane but less readily than carbon

monoxide. Nitrogen is found in the atmosphere of a planet only if nitrogen ice is found on the surface of that planet. A mixture of nitrogen, carbon monoxide, and methane is characteristic of the substances from which the Solar System formed.

Sources: PrcpTest 24,9ectlon 3, Questlon 78


fuepTest
PrepTest

22, Sectlon 2, Questlon

t9

Preplest 22, Sectlon 4, Quastlan 22

21, Sectlon 2, Questlon 19

224

PDF compression, OCR, web optimization using a watermarked evaluation copy of CVISION PDFCompressor

Assumption Questions
89.

Eva:

A "smart highway" system should be installed, one that would monitor areawide traffic

91.

Cafeteria

patterns and communicate with computers in vehicles or with programmable highway signs to give drivers information about traffic congestion and alternate routes. Such a system,
we can infer, would result in improved traffic

flow in and around cities that would do more than improve drivers'tempersi it would decrease the considerable loss of money and productivity that now results from traffic
congestion.

patron: The apples sold in this cafeteria are greasy. The cashier told me that the apples are in that condition when they are delivered to the cafeteria and that the cafeteria does not wash the apples it sells. Most fruit is sprayed with dangerous pesticides before it is harvested, and is dangerous until it is washed. Clearly, the cafeteria is selling pesticidecovered fruit, thereby endangering its patrons.

Which one of the following is an assumption on


which the argument depends?

Luis:

There are already traffic rePorts on the radio. Why would a "smart highway''system be any better?

(A) (B) (C) (D) (E)


92.

Eva's argument depends on the assumption that

(A) (B) (C) (D) (E)

on "smart highways" there would not be the breakdowns of vehicles that currently cause traffic congestion trafEc lights, if coordinated by the system,

would assure a free flow of traffic traffic flow in and around cities is not now so congested that significant improvement is impossible the type of equipment used in "smart highway'' systems would vary from one city to another older vehicles could not be fitted with equipment to receive signals sent by a "smart highway" system

The apples that the cafeteria sells are not thoroughly washed after harvest but before reaching the cafeteria, Most pesticides that are sprayed on fruit before harvest leave a greasy residue on the fruit. Many of the cafeteria's patrons are unaware that the cafeteria does not wash the apples it sells. Only pesticides that leave a greasy residue on fruit can be washed off. Fruits other than apples also arrive at the cafeteria in a greasy condition.

* * * *

Ann will either take a leave of absence from Technocomp and return in a year or else she will quit her job there; but she would not do either one unless she were offered a one-year teaching fellowship at a prestigious university. Technocomp will allow her to take a leave of absence if it does not find out that she has been offered the fellowship, but not otherwise. Therefore, Ann will quit her job at Technocomp only if Technocomp finds out she has been offered the fellowship.

Nuclear reactors are sometimes built in "geologically quiet" regions, so called by geologists because such regions are distant from plate boundaries and contain only minor faults. Since no minor fault in a geologically quiet region produces an earthquake more often than once in any given 100,000-year period, it follows that of all potentiai nuclear reactor sites in such a region, those that are least likely to be

F
oe IT rlt ql
I

c
o
TI = I )

7
o

struckby an earthquake are ones located near a fault that has produced an earthquake within living
memory.

!f t,|
ort

Which one of the following is an assumption on


which the argument depends?

(A) (B) (C) (D)

Geologically quiet regions are the least dangerous regions in which to build nuclear
reactors.

Which one of the following, if assumed, allows the


conclusion above to be properly drawn?

(A) (B) (C) (D) (E)

Technocomp will find out about Ann being offered the fellowship only if someone informs on her. The reason Ann wants the fellowship is so she can quit her job at Technocomp. Technocomp does not allow any of its employees to take a leave of absence in order to work for one of its competitors, Ann will take a leave of absence if Technocomp allows her to take a leave of absence. Ann would be offered the fellowship only if she quit her job at Technocomp.

For any potential nuclear reactor site, the likelihood of being struck by an earthquake is the primary determinant of site safety. In a geologically quiet region, every potential
nuclear reactor site is near at least one minor

fault. Nuclear reactors that are located in geologically quiet regions are built to withstand at least one but not necessarily more than one earthquake of minor to
moderate force. Earthquake faults in geologically quiet regions produce earthquakes at least once in 100,000
years.

(E)

Sources; PrcpTest 21, Sectlon 3, Questlon PrepTest 27, Sectlon 2, Quastlon 20 Prcplest 17, Sectlon 2, Questlone t0 and 27

22s
PDF compression, OCR, web optimization using a watermarked evaluation copy of CVISION PDFCompressor

ISAT Mostery Practice

93.

Magazine editor: I know that some of our regular advertisers have been pressuring us to give favorable mention to their products in our articles, but they should realize that for us to yield to their wishes would actually be against their interests. To remain an effective advertising vehicle we must have loyal readership, and we would soon lose that readership if our readers suspect that our editorial integrity has been compromised by pandering to advertisers. Advertising-sales

94,

Even the earliest known species of land animals,

known from fossils dating from the late Silurian period, 400 million years ago, show highly evolved adaptations to life on land. Since neither aquatic nor amphibious animals exhibit these adaptations, early species of land aaimals must have evolved very rapidly after leaving an aquatic environment. Which one of the following is an assumption on which the argument depends?

(A) (B) (C) (D) (E)

* * * *
ElD
I

director: You underestimate the sophistication of our readers. They recognize that the advertisements we carry are not articles, so their response to the
advertisements has never depended on their opinion of the editorial integrity of the magazine as a whole.

The magazine editor's argument assumes which one of the following?

(A)
F rI

E
I

o It G o

(B) (C) (D) (B)

A magazine editor should never be in{luenced in the performance of his or her professional duties by the wishes of the companies that regularly advertise in the magazine.
The magazine cannot give any favorable mention in its articles to its regular advertisers without compromising its

Known fossils of early land animals include fossils of animals that lived relatively soon after the first emergence of land animals. Fossils from the late Silurian period represent only a small number of the animal species that were alive at that time. No plants were established on land before the late Silurian period. No present-day species of aquatic animal is descended from a species of animal that once lived on land. All animals alive in the late Silurian period lived either exclusively on land or exclusivelv

in the water.
95.

tJ rI bo

IE

reputation for editorial integrity. Favorable mention of their products in the magazine's articles is of less value to the advertisers than is the continued effectiveness of the magazine as an advertising vehicle. Giving favorable mention to a product in a
magazine article is a more effective form of advertisement than is an explicit advertisement for the product in the same magazine, Carrying paid advertisements can never pose

@ @

A poor farmer was fond of telling his children: "In this world, you are either rich or poor, and you are either honest or dishonest. All poor farmers are honest. Therefore, all rich farmers are dishonest."
The farmer's conclusion is properly drawn if the

argument assumes that

o J

(A) (B) (C) (D) (E)

every honest farmer is poor every honest person is a farmer everyone who is dishonest is a rich farmer eYeryone who is poor is honest every poor person is a farmer

any threat to the magazine's reputation for editorial integrity nor to the loyalty of its readership.

Sources; PrepTest 17, Sectlon 2, Questlon 23

!7, Sectlon 3, Question 2 PrcpTest 9t Sectlon 2, Questlon 23


PrepTest

226
PDF compression, OCR, web optimization using a watermarked evaluation copy of CVISION PDFCompressor

Assumption Questions 96.


Medical research findings are customarily not made public prior to their publication in a medical journal that has had them reviewed by a panel of experts in a process called peer review. It is claimed that this practice delays public access to potentially beneficial information that, in extreme instances, could save lives. Yet prepublication peer review is the only way to prevent erroneous and therefore potentially haimful information from reaching a public that is ill equipped to evaluate medical claims on its own. Therefore, waiting until a medical journal has published the research findings that have passed peer ieview is the price that must be paid to protect the public from making decisions based on possibly
substandard research' The argument assumes that

97. Marcusl

For most ethical dilemmas the journalist is likely to face, traditional journalistic ethics is
clear, adequate, and essentially correct. For example, when journalists have uncovered newsworthy information, they should go to press with it as soon as possible. No delay

motivated by the journalists'personal or


professional interests is permissible.

Anita:

Well, Marcus, of course interesting and

important information should be brought


before the public-that is a journalistt job. But in the typical case, where a journalist has some information but is in a quandary about whether it is yet important or "newsworthy," this guidance is inadequate.

(A) (B) (C) (D) (E)

unless medical research findings are brought

to peer review by a medical journal, peer review will not occur anyone who does not setve on a medical review panel does not have the necessary knowledge and expertise to evaluate medical research findings the general public does not have access to the medical iournals in which research findings are published all medical research findings are subjected to prepublication Peer review
peer review panels are sometimes subject to political and professional pressures that can make their judgments less than impartial

In order to conclude properly from Anita's statements that Marcus'general claim about traditional journalistic ethics is incorrect, it would have to be assumed that

(A) (B) (C) (D) (E)

* * * *

whether a piece of information is or is not newsworthy can raise ethical dilemmas for journalists there are circumstances in which it would be ethically wrong for a journalist to go to press with legitimately acquired, newsworthy

information
the most serious professional dilemmas that a

0 gq II a

g.

journalist is likely to face are not ethical


dilemmas there are no ethical dilemmas that
a

journalist

7 o
oq
Of o o = 3
II

is likely to face that would not be conclusively

resolved by an adequate system ofjournalistic ethics for a system ofjournalistic ethics to be adequate it must be able to provide guidance in every case in which a journalist must make a professional decision

Sources.' PrepTest PrepTest

9, Sectlon 2, Queetlon 25 9, Section 4, Questlon t9

227

PDF compression, OCR, web optimization using a watermarked evaluation copy of CVISION PDFCompressor

98. Mayor

of Plainwills In order to help the economy of Plainsville, I am using some of our tax revenues to help bring a major highway through the town and thereby attract new business to Plainsville.

Citizens'group: You must have interests o&er than our economy in mind. If you were really interested in helping our economy, you would instead aUocate the revenues to building a new business park, since it would bring in twice the business that your highwaY would.
The argument by the citizens'group relies on which one of the following assumptions?

* * * *
BO
II

(A) (B) (C) (D)

tr

t:

(E)

Plainsville presendy has no major highways running through it. The mayor accePts that a new business park would bring in more new business than would the new highway. The new highway would have no benefits for Plainsville other than attracting new business. The mayor is required to get aPproval for all tax revenue allocation plans from the city council. Plainsvillet economy will not be helped unless a new business park of the sort envisioned by
the citizens'group is built.

{a

o sl
l:

GI

-ts

tJ b0

Source: Prepiest

5, Sectlon

t,

Questlon 3

228
PDF compression, OCR, web optimization using a watermarked evaluation copy of CVISION PDFCompressor

Você também pode gostar